Partitioning the positive integers into finitely many arithmetic progressionsCovering the primes by 3-term...



Partitioning the positive integers into finitely many arithmetic progressions


Covering the primes by 3-term APs ?Is a “non-analytic” proof of Dirichlet's theorem on primes known or possible?Finitely many arithmetic progressionsThe Green-Tao theorem and positive binary quadratic formsWhat is the shortest route to Roth's theorem?Arithmetic progressions in power sequencesWhat are the limits of the Erdős-Rankin method for covering intervals by arithmetic progressions?Intersection of two arithmetic progressionsAbout consecutive integers covered by arithmetic progressionsProduct of arithmetic progressions













2












$begingroup$


From Bóna's A Walk through Combinatorics:




Prove or disprove that if we partition the positive integers into finitely many arithmetic progressions then there will be at least one arithmetic progression whose difference and initial term are equal.




A variant of this problems asks to prove that there will be at least one arithmetic progression whose difference divides the initial term, which has an elementary proof. I suspect the same arithmetic progression will have the initial term equal to the difference. However, I cannot prove it.










share|cite|improve this question









New contributor




John is a new contributor to this site. Take care in asking for clarification, commenting, and answering.
Check out our Code of Conduct.







$endgroup$








  • 2




    $begingroup$
    What is the origin of this question? Usually if you ask to "prove" something you should have some strong evidence that the proof exists.
    $endgroup$
    – Fedor Petrov
    13 hours ago






  • 1




    $begingroup$
    Sorry I should’ve asked for counter example also. I have edited the question now. The source is the variant mentioned below the main question which is an exercise from “A Walk Through Combinatorics”.
    $endgroup$
    – John
    12 hours ago










  • $begingroup$
    Counter example where there are an infinite number of infinite arithmetic progressions and NO progression has the difference equal to the initial term. for k from 1 to inf { S(k) = 2^k * n + 2 ^ (k-1) for n from 0 to inf } S(1) is odd integers; S(2) is 2, 6, 10 - twice an odd integer; and continues for powers of 2. In all cases the initial term is the difference / 2.
    $endgroup$
    – Judah Greenblatt
    6 hours ago
















2












$begingroup$


From Bóna's A Walk through Combinatorics:




Prove or disprove that if we partition the positive integers into finitely many arithmetic progressions then there will be at least one arithmetic progression whose difference and initial term are equal.




A variant of this problems asks to prove that there will be at least one arithmetic progression whose difference divides the initial term, which has an elementary proof. I suspect the same arithmetic progression will have the initial term equal to the difference. However, I cannot prove it.










share|cite|improve this question









New contributor




John is a new contributor to this site. Take care in asking for clarification, commenting, and answering.
Check out our Code of Conduct.







$endgroup$








  • 2




    $begingroup$
    What is the origin of this question? Usually if you ask to "prove" something you should have some strong evidence that the proof exists.
    $endgroup$
    – Fedor Petrov
    13 hours ago






  • 1




    $begingroup$
    Sorry I should’ve asked for counter example also. I have edited the question now. The source is the variant mentioned below the main question which is an exercise from “A Walk Through Combinatorics”.
    $endgroup$
    – John
    12 hours ago










  • $begingroup$
    Counter example where there are an infinite number of infinite arithmetic progressions and NO progression has the difference equal to the initial term. for k from 1 to inf { S(k) = 2^k * n + 2 ^ (k-1) for n from 0 to inf } S(1) is odd integers; S(2) is 2, 6, 10 - twice an odd integer; and continues for powers of 2. In all cases the initial term is the difference / 2.
    $endgroup$
    – Judah Greenblatt
    6 hours ago














2












2








2





$begingroup$


From Bóna's A Walk through Combinatorics:




Prove or disprove that if we partition the positive integers into finitely many arithmetic progressions then there will be at least one arithmetic progression whose difference and initial term are equal.




A variant of this problems asks to prove that there will be at least one arithmetic progression whose difference divides the initial term, which has an elementary proof. I suspect the same arithmetic progression will have the initial term equal to the difference. However, I cannot prove it.










share|cite|improve this question









New contributor




John is a new contributor to this site. Take care in asking for clarification, commenting, and answering.
Check out our Code of Conduct.







$endgroup$




From Bóna's A Walk through Combinatorics:




Prove or disprove that if we partition the positive integers into finitely many arithmetic progressions then there will be at least one arithmetic progression whose difference and initial term are equal.




A variant of this problems asks to prove that there will be at least one arithmetic progression whose difference divides the initial term, which has an elementary proof. I suspect the same arithmetic progression will have the initial term equal to the difference. However, I cannot prove it.







arithmetic-progression elementary-proofs






share|cite|improve this question









New contributor




John is a new contributor to this site. Take care in asking for clarification, commenting, and answering.
Check out our Code of Conduct.











share|cite|improve this question









New contributor




John is a new contributor to this site. Take care in asking for clarification, commenting, and answering.
Check out our Code of Conduct.









share|cite|improve this question




share|cite|improve this question








edited 6 hours ago









Rodrigo de Azevedo

1,8422719




1,8422719






New contributor




John is a new contributor to this site. Take care in asking for clarification, commenting, and answering.
Check out our Code of Conduct.









asked 14 hours ago









JohnJohn

194




194




New contributor




John is a new contributor to this site. Take care in asking for clarification, commenting, and answering.
Check out our Code of Conduct.





New contributor





John is a new contributor to this site. Take care in asking for clarification, commenting, and answering.
Check out our Code of Conduct.






John is a new contributor to this site. Take care in asking for clarification, commenting, and answering.
Check out our Code of Conduct.








  • 2




    $begingroup$
    What is the origin of this question? Usually if you ask to "prove" something you should have some strong evidence that the proof exists.
    $endgroup$
    – Fedor Petrov
    13 hours ago






  • 1




    $begingroup$
    Sorry I should’ve asked for counter example also. I have edited the question now. The source is the variant mentioned below the main question which is an exercise from “A Walk Through Combinatorics”.
    $endgroup$
    – John
    12 hours ago










  • $begingroup$
    Counter example where there are an infinite number of infinite arithmetic progressions and NO progression has the difference equal to the initial term. for k from 1 to inf { S(k) = 2^k * n + 2 ^ (k-1) for n from 0 to inf } S(1) is odd integers; S(2) is 2, 6, 10 - twice an odd integer; and continues for powers of 2. In all cases the initial term is the difference / 2.
    $endgroup$
    – Judah Greenblatt
    6 hours ago














  • 2




    $begingroup$
    What is the origin of this question? Usually if you ask to "prove" something you should have some strong evidence that the proof exists.
    $endgroup$
    – Fedor Petrov
    13 hours ago






  • 1




    $begingroup$
    Sorry I should’ve asked for counter example also. I have edited the question now. The source is the variant mentioned below the main question which is an exercise from “A Walk Through Combinatorics”.
    $endgroup$
    – John
    12 hours ago










  • $begingroup$
    Counter example where there are an infinite number of infinite arithmetic progressions and NO progression has the difference equal to the initial term. for k from 1 to inf { S(k) = 2^k * n + 2 ^ (k-1) for n from 0 to inf } S(1) is odd integers; S(2) is 2, 6, 10 - twice an odd integer; and continues for powers of 2. In all cases the initial term is the difference / 2.
    $endgroup$
    – Judah Greenblatt
    6 hours ago








2




2




$begingroup$
What is the origin of this question? Usually if you ask to "prove" something you should have some strong evidence that the proof exists.
$endgroup$
– Fedor Petrov
13 hours ago




$begingroup$
What is the origin of this question? Usually if you ask to "prove" something you should have some strong evidence that the proof exists.
$endgroup$
– Fedor Petrov
13 hours ago




1




1




$begingroup$
Sorry I should’ve asked for counter example also. I have edited the question now. The source is the variant mentioned below the main question which is an exercise from “A Walk Through Combinatorics”.
$endgroup$
– John
12 hours ago




$begingroup$
Sorry I should’ve asked for counter example also. I have edited the question now. The source is the variant mentioned below the main question which is an exercise from “A Walk Through Combinatorics”.
$endgroup$
– John
12 hours ago












$begingroup$
Counter example where there are an infinite number of infinite arithmetic progressions and NO progression has the difference equal to the initial term. for k from 1 to inf { S(k) = 2^k * n + 2 ^ (k-1) for n from 0 to inf } S(1) is odd integers; S(2) is 2, 6, 10 - twice an odd integer; and continues for powers of 2. In all cases the initial term is the difference / 2.
$endgroup$
– Judah Greenblatt
6 hours ago




$begingroup$
Counter example where there are an infinite number of infinite arithmetic progressions and NO progression has the difference equal to the initial term. for k from 1 to inf { S(k) = 2^k * n + 2 ^ (k-1) for n from 0 to inf } S(1) is odd integers; S(2) is 2, 6, 10 - twice an odd integer; and continues for powers of 2. In all cases the initial term is the difference / 2.
$endgroup$
– Judah Greenblatt
6 hours ago










1 Answer
1






active

oldest

votes


















7












$begingroup$

Any progression (if they are all infinite, of course, but otherwise the statement is clearly wrong) should have its initial term $a$ not greater than the difference $d$. Indeed, if $a>d$, and $a-d$ is covered by another progression $P$ with difference $d_1$, then $a-d+dd_1$ is covered twice --- a contradiction. Therefore if $a$ is divisible by $d$, we must have $a=d$. And such $a$ exists as you noted in OP (a short proof: take a progression which contains the product of all differences).






share|cite|improve this answer











$endgroup$













    Your Answer





    StackExchange.ifUsing("editor", function () {
    return StackExchange.using("mathjaxEditing", function () {
    StackExchange.MarkdownEditor.creationCallbacks.add(function (editor, postfix) {
    StackExchange.mathjaxEditing.prepareWmdForMathJax(editor, postfix, [["$", "$"], ["\\(","\\)"]]);
    });
    });
    }, "mathjax-editing");

    StackExchange.ready(function() {
    var channelOptions = {
    tags: "".split(" "),
    id: "504"
    };
    initTagRenderer("".split(" "), "".split(" "), channelOptions);

    StackExchange.using("externalEditor", function() {
    // Have to fire editor after snippets, if snippets enabled
    if (StackExchange.settings.snippets.snippetsEnabled) {
    StackExchange.using("snippets", function() {
    createEditor();
    });
    }
    else {
    createEditor();
    }
    });

    function createEditor() {
    StackExchange.prepareEditor({
    heartbeatType: 'answer',
    autoActivateHeartbeat: false,
    convertImagesToLinks: true,
    noModals: true,
    showLowRepImageUploadWarning: true,
    reputationToPostImages: 10,
    bindNavPrevention: true,
    postfix: "",
    imageUploader: {
    brandingHtml: "Powered by u003ca class="icon-imgur-white" href="https://imgur.com/"u003eu003c/au003e",
    contentPolicyHtml: "User contributions licensed under u003ca href="https://creativecommons.org/licenses/by-sa/3.0/"u003ecc by-sa 3.0 with attribution requiredu003c/au003e u003ca href="https://stackoverflow.com/legal/content-policy"u003e(content policy)u003c/au003e",
    allowUrls: true
    },
    noCode: true, onDemand: true,
    discardSelector: ".discard-answer"
    ,immediatelyShowMarkdownHelp:true
    });


    }
    });






    John is a new contributor. Be nice, and check out our Code of Conduct.










    draft saved

    draft discarded


















    StackExchange.ready(
    function () {
    StackExchange.openid.initPostLogin('.new-post-login', 'https%3a%2f%2fmathoverflow.net%2fquestions%2f324088%2fpartitioning-the-positive-integers-into-finitely-many-arithmetic-progressions%23new-answer', 'question_page');
    }
    );

    Post as a guest















    Required, but never shown

























    1 Answer
    1






    active

    oldest

    votes








    1 Answer
    1






    active

    oldest

    votes









    active

    oldest

    votes






    active

    oldest

    votes









    7












    $begingroup$

    Any progression (if they are all infinite, of course, but otherwise the statement is clearly wrong) should have its initial term $a$ not greater than the difference $d$. Indeed, if $a>d$, and $a-d$ is covered by another progression $P$ with difference $d_1$, then $a-d+dd_1$ is covered twice --- a contradiction. Therefore if $a$ is divisible by $d$, we must have $a=d$. And such $a$ exists as you noted in OP (a short proof: take a progression which contains the product of all differences).






    share|cite|improve this answer











    $endgroup$


















      7












      $begingroup$

      Any progression (if they are all infinite, of course, but otherwise the statement is clearly wrong) should have its initial term $a$ not greater than the difference $d$. Indeed, if $a>d$, and $a-d$ is covered by another progression $P$ with difference $d_1$, then $a-d+dd_1$ is covered twice --- a contradiction. Therefore if $a$ is divisible by $d$, we must have $a=d$. And such $a$ exists as you noted in OP (a short proof: take a progression which contains the product of all differences).






      share|cite|improve this answer











      $endgroup$
















        7












        7








        7





        $begingroup$

        Any progression (if they are all infinite, of course, but otherwise the statement is clearly wrong) should have its initial term $a$ not greater than the difference $d$. Indeed, if $a>d$, and $a-d$ is covered by another progression $P$ with difference $d_1$, then $a-d+dd_1$ is covered twice --- a contradiction. Therefore if $a$ is divisible by $d$, we must have $a=d$. And such $a$ exists as you noted in OP (a short proof: take a progression which contains the product of all differences).






        share|cite|improve this answer











        $endgroup$



        Any progression (if they are all infinite, of course, but otherwise the statement is clearly wrong) should have its initial term $a$ not greater than the difference $d$. Indeed, if $a>d$, and $a-d$ is covered by another progression $P$ with difference $d_1$, then $a-d+dd_1$ is covered twice --- a contradiction. Therefore if $a$ is divisible by $d$, we must have $a=d$. And such $a$ exists as you noted in OP (a short proof: take a progression which contains the product of all differences).







        share|cite|improve this answer














        share|cite|improve this answer



        share|cite|improve this answer








        edited 8 hours ago

























        answered 12 hours ago









        Fedor PetrovFedor Petrov

        49.6k5114230




        49.6k5114230






















            John is a new contributor. Be nice, and check out our Code of Conduct.










            draft saved

            draft discarded


















            John is a new contributor. Be nice, and check out our Code of Conduct.













            John is a new contributor. Be nice, and check out our Code of Conduct.












            John is a new contributor. Be nice, and check out our Code of Conduct.
















            Thanks for contributing an answer to MathOverflow!


            • Please be sure to answer the question. Provide details and share your research!

            But avoid



            • Asking for help, clarification, or responding to other answers.

            • Making statements based on opinion; back them up with references or personal experience.


            Use MathJax to format equations. MathJax reference.


            To learn more, see our tips on writing great answers.




            draft saved


            draft discarded














            StackExchange.ready(
            function () {
            StackExchange.openid.initPostLogin('.new-post-login', 'https%3a%2f%2fmathoverflow.net%2fquestions%2f324088%2fpartitioning-the-positive-integers-into-finitely-many-arithmetic-progressions%23new-answer', 'question_page');
            }
            );

            Post as a guest















            Required, but never shown





















































            Required, but never shown














            Required, but never shown












            Required, but never shown







            Required, but never shown

































            Required, but never shown














            Required, but never shown












            Required, but never shown







            Required, but never shown







            Popular posts from this blog

            Why do type traits not work with types in namespace scope?What are POD types in C++?Why can templates only be...

            Will tsunami waves travel forever if there was no land?Why do tsunami waves begin with the water flowing away...

            Should I use Docker or LXD?How to cache (more) data on SSD/RAM to avoid spin up?Unable to get Windows File...